Planning for Retirement Needs Flashcards

CFP Class

You may prefer our related Brainscape-certified flashcards:
1
Q

T/ F

A target-benefit pension plan is a money-purchase pension plan with a funding formula that actuarially considers age and salary.

A

True

How well did you know this?
1
Not at all
2
3
4
5
Perfectly
2
Q

T/ F

The plan sponsor of a target-benefit pension plan guarantees that the participant will receive an amount, expected to be the “target benefit” amount, at his retirement

A

False

How well did you know this?
1
Not at all
2
3
4
5
Perfectly
3
Q

Plan participants of a target-benefit pension plan can choose to invest up to what amount of account assets into employer stock?

A

No qualified plan may invest more than 10% of its assets into employer securities.

How well did you know this?
1
Not at all
2
3
4
5
Perfectly
4
Q

T/ F

The plan sponsor is required to make an annual contribution to a money-purchase pension plan.

A

True

How well did you know this?
1
Not at all
2
3
4
5
Perfectly
5
Q

T/ F

The excess earnings of a money-purchase pension plan are returned to the plan sponsor.

A

False - The participant of a money-purchase pension plan bears all the investment risk (and benefit)

How well did you know this?
1
Not at all
2
3
4
5
Perfectly
6
Q

T/ F

A cash-balance pension plan does not have individual separate accounts for each participant.

A

True - a cash-balance pension plan consists of a commingled account that must be equal to the actuarial equivalent of the present value of the account expected future benefits that will be paid from the cash-balance pension plan.

How well did you know this?
1
Not at all
2
3
4
5
Perfectly
7
Q

T/ F

A defined-benefit plan promises a contribution to a hypothetical account each year for a plan participant.

A

False - a defined-benefit plan promises a defined benefit at the participant’s retirement. If the funding requirements are met with plan earnings, a contribution is not required.

How well did you know this?
1
Not at all
2
3
4
5
Perfectly
8
Q

Susan’s new employer is expecting to contribute $5,000 annually toward her pension account. She also has an option to include a whole-life insurance policy within that account. Given her current age, the level annual premiums would run $10 per $1,000 of death benefit.

What is the maximum policy death benefit that her employer can purchase for Susan?

A

$250,000

Susan’s pension account starts out empty but will gain $5,000 annually of employer contributions. According to the 25%/50% test, the aggregate of whole-life premiums can’t exceed 50% of employer contributions, or $2,500 annually.

Since this level premium policy costs $10 annually per $1,000 of death benefit, the maximum benefit is ($2,500/ $10) x $1,000 = $250,000.

How well did you know this?
1
Not at all
2
3
4
5
Perfectly
9
Q

What makes a “highly compensated employee”?

A
  1. > 5% owner of the company
  2. Compensation > $135,000
How well did you know this?
1
Not at all
2
3
4
5
Perfectly
10
Q

What is the Ratio Percentage Test?

What is the ratio test requirement?

How do you calculate it?

A

The Ratio percentage test compares the percentage of covered non-highly compensated to highly compensated employees.
So it’s testing to make sure enough non-highly compensate employees are enrolled in the plan.

The requirement is 70% - so 70% of NHC employees must be covered in relation to covered HC employees.

You take the % of NHC employees and divide by the % of HC employees.

How well did you know this?
1
Not at all
2
3
4
5
Perfectly
11
Q

What is the average benefits percentage test?

What is the test requirement?

How do you calculate it?

A

The average benefits percentage test tests to make sure NHC employees are taking advantage of the plan to a certain extent compared to HC employees.

The requirement is that NHC employees must be taking 70% advantage compared to HC employees.

To calculate, you divide NHC % by HC %.

If the average accrued benefit for HC employees is 4% and the average for NHC employees is 1.5%, you do 1.5%/ 4% = 37.5% (FAIL).

If HC employees are accruing 4%, NHC employees must be accruing at least 2.8%. (2.8%/ 4% = 70%)

How well did you know this?
1
Not at all
2
3
4
5
Perfectly
12
Q

What is the 50/40 rule?

A

An ERISA rule which tells you how many employees must be covered in your defined-benefit plan.

It’s a lesser of either/ or rule.

EITHER 50 employees OR 40% of all eligible employees.

EX.
A company has 100 employees so EITHER 50 employees must be covered OR 40% of 100 = 40. The lesser number is 40 - so 40 employees must be covered in this plan according to the 50/ 40 rule.

How well did you know this?
1
Not at all
2
3
4
5
Perfectly
13
Q

What is the least generous graduated-vesting schedule of a 401k?

A

2- to 6-year graduated -vesting schedule

After 6 years the employee will be entirely vested.

How well did you know this?
1
Not at all
2
3
4
5
Perfectly
14
Q

When calculating wage replacement, what is one annual expense that is never specifically called for and what % of your salary is it?

A

Payroll Taxes. 7.65%

Whenever calculating wage replacement, deduct all listed items PLUS 7.65% of the person’s income.

Ex.
100k salary
- 12k/ yr mortgage
- 15k/ yr savings
- 7,650 payroll taxes (7.65% of 100k)

= 65.35% wage replacement

How well did you know this?
1
Not at all
2
3
4
5
Perfectly
15
Q

T/ F

K-12 expenses are considered qualifying expenses under IRC 127 educational assistance program.

A

False

Qualifying expenses are for undergraduate and graduate programs as well as student loan payments.

How well did you know this?
1
Not at all
2
3
4
5
Perfectly
16
Q

T/ F

Under IRC 127 educational assistance program, up to $5,250 of expenses may be excluded from the employee’s gross income.

A

True

How well did you know this?
1
Not at all
2
3
4
5
Perfectly
17
Q

T/ F

Parking provided to employees, paid by the employer is deductible for the employee and not included in employee’s gross income.

A

False.

It will not be included in the employee’s gross income but it IS NOT deductible to the employer.

How well did you know this?
1
Not at all
2
3
4
5
Perfectly
18
Q

T/ F

Regarding Voluntary Employee Beneficiary Associations (VEBAs), the employee may be provided retirement and miscellaneous fringe benefits.

A

False

Employees may not be provided retirement or misc fringe benefits.

How well did you know this?
1
Not at all
2
3
4
5
Perfectly
19
Q

T/F

Regarding Voluntary Employee Beneficiary Associations (VEBAs), the employer avoids taxation on the investment income earned on contributions.

A

True

How well did you know this?
1
Not at all
2
3
4
5
Perfectly
20
Q

T/F

Regarding Voluntary Employee Beneficiary Associations (VEBAs), noncurrent employees may become members of a VEBA.

A

True

Note: the number of noncurrent employees that may become members of a VEBA cannot exceed 10% of the total membership.

If there are 100 employees, no more than 10 may be non current employees.

How well did you know this?
1
Not at all
2
3
4
5
Perfectly
21
Q

T/F

Most cafeteria plans offer a pension plan or some type of deferred-comp plan

A

False

401k plans are the only type of retirement plan that may be offered through a cafeteria plan.

How well did you know this?
1
Not at all
2
3
4
5
Perfectly
22
Q

T/F

Under a cafeteria plan, a family can write its own benefit plan by selecting among available options.

A

True

How well did you know this?
1
Not at all
2
3
4
5
Perfectly
23
Q

What is the income phaseout for adoption assistance?

A

$263,410

How well did you know this?
1
Not at all
2
3
4
5
Perfectly
24
Q

If highly compensated employees receive a discriminatory fringe benefit, do they have to report it on their taxes? If so how much?

A

Yes. They need to the entire amount on their taxes.

How well did you know this?
1
Not at all
2
3
4
5
Perfectly
25
Q

T/F

Participants of a secular trust have no security against an employer’s bankruptcy.

A

False - Secular trusts DO protect against bankruptcy. Secular trust assets are owned by the trust, not the company so if the company goes bankrupt, the trust does not.

How well did you know this?
1
Not at all
2
3
4
5
Perfectly
26
Q

T/F

Participants of a secular trust have security against an employer’s change of heart.

A

True

How well did you know this?
1
Not at all
2
3
4
5
Perfectly
27
Q

How are non-qualified stock options taxed? NQSOs

A

The gain is taxed as W-2 income.

Ex. You are granted NQSOs with a strike price of $100 and excesise them at $150. You realize $50/ share of W2 income.

How well did you know this?
1
Not at all
2
3
4
5
Perfectly
28
Q

How are Incentive Stock Options, ISOs, taxed?

A

At exercise, there is no taxable event. You realize the cost basis of the exercise price of the option. Only when you sell the stock is there a taxable event.

How well did you know this?
1
Not at all
2
3
4
5
Perfectly
29
Q

Why would you make an 83(b) election on restricted stock?

A

You can claim the cost basis of the stock before you actually own it (before it vests). So if you sell a year after you file 83(b) you can receive long term cap gain tax treatment or, if the stock goes down, you can claim a loss.

How well did you know this?
1
Not at all
2
3
4
5
Perfectly
30
Q

T/ F

SIMPLE plans can be either contributory or noncontributory plans, whereas SEP plans are always noncontributory.

A

True

How well did you know this?
1
Not at all
2
3
4
5
Perfectly
31
Q

A SEP plan can be established by employers who employ more than 100 employees who earn $5,000 or more during the preceding calendar year.

A

True - there is a 100 employee limit on SIMPLE plans though.

How well did you know this?
1
Not at all
2
3
4
5
Perfectly
32
Q

T/ F

All 403(b) plans must pass the ADP test.

A

False, only contributory 403(b) plans do.

How well did you know this?
1
Not at all
2
3
4
5
Perfectly
33
Q

Do SEP IRAs allow for employee salary contributions?

A

No

How well did you know this?
1
Not at all
2
3
4
5
Perfectly
34
Q

What is contribution limit for an employee in a SIMPLE plan?

A

2023: 15,500 + $3500 if you’re over 50
2022: $14,000 + $3,000 if you’re over 50

plus employer match

How well did you know this?
1
Not at all
2
3
4
5
Perfectly
35
Q

When / who needs to file a Form 5500 EZ?

A

One participant plan holders with > $250,000 in the account.

How well did you know this?
1
Not at all
2
3
4
5
Perfectly
36
Q

Regarding divorce approach, what is the separate interest approach?

A

Calls for splitting a retirement account into two separate accounts.

37
Q

Are inherited Roth accounts subject to RMDs?

A

Yes

38
Q

What is the maximum vesting time for a cash-balance plan?

A

3 years

39
Q

T/ F

in-service withdrawals for employees under 59.5 years old is a characteristic of a pension plan.

A

False

40
Q

T/F

The excludable amount formula is a common defined-benefit plan funding formula.

A

False

41
Q

T/ F

Pension plans have a mandatory funding requirement.

A

True

42
Q

T/F

The flat amount formula is a common defined-benefit plan funding formula.

A

True

43
Q

T/F

The flat percentage formula is a common defined-benefit plan funding formula.

A

True

44
Q

T/F

The plan sponsor of a money-purchase plan generally bears the investment risk of the plan assets.

A

False. The participant bears the risk and reward of he plan.

45
Q

T/F

A plan sponsor with fluctuating cash flows would likely adopt a money-purchase pension plan.

A

False. An employer with fluctuating cash flows would not choose a money-purchase plan due to mandatory annual funding requirements.

46
Q

T/ F

A cash-balance pension plan usually benefits older employees the most.

A

False. Cash-balance pension plans usually benefit younger parts because of the guaranteed contribution rate and guaranteed earnings rate.

47
Q

Which of the following statements are true?

I. All Pension Plans are required to participate in Pension Benefit Guarantee Corporation (PBGC)
II. Cash Balance Plans with 50 Participants must participate in the PBGC

A

II only

48
Q

Who needs to participate in Pension Benefit Guarantee Corporation (PBGC)?

A

Defined benefit plans with more than 25 participants.

49
Q

What are the employee contribution limits to a 401k?

A

$22,500 or $30,000 if you’re over 50 years old

50
Q

T/F

A negative election is a provision whereby the employee is deemed to have elected a specific deferral unless the employee specifically elects out of such election in writing.

A

True

51
Q

T/F

When an employer includes a negative election in its qualified plan, the employer must also provide 100% immediate vesting.

A

False

52
Q

T/F

Profit-sharing plans may not offer in-service withdrawals.

A

False. profit-sharing plans MAY offer in service withdrawals.

53
Q

T/F

Profit-sharing plans allow annual employer contributions up to 25 percent of the employer’s covered compensation.

A

True. Profit sharing plans allow annual contributions of up to 25 percent of covered compensation.

54
Q

Which of the following statements concerning profit-sharing plans is (are) correct?

I. The employer can choose from a number of different allocation formulas.
II. Profit-sharing plans are popular because contributions may be made on a discretionary basis.

A

Both I and II

55
Q

Which of the following are requirements for a qualified stock-bonus plan?

I. Participants must have pass-through voting rights for stock held by the plan.
II. Participants must have the right to demand employer securities at a distribution, even if the plan sponsor is a closely held corporation.

A

Both I and II

56
Q

One of the disadvantages of an ESOP is that the stock is an undiversified investment portfolio. Which of the following is correct?

I. An employee, aged 55 or older, who has completed 5 years of participation in an ESOP may require that up to 25 percent of the account balance be diversified.
II. An employee who receives corporate stock from an ESOP in a series of periodic distributions over 3 years may enjoy net unrealized appreciation treatment at the time of each distribution.

A

Neither I nor II

I is wrong because the time required to request diversification is 10 years.

II is wrong because NUA is only available in case when all the account assets are distributed in 1 tax year.

57
Q

T/ F

To qualify for non-recognition of gain treatment, the stock sold to the ESOP must be common or convertible preferred stock and must have been owned by the seller for no more than three years prior to the sale.

A

False.

The stock sold must have been owned by the seller for at least 3 years, not “no more than” three years.

58
Q

T/ F

To qualify for non-recognition of gain treatment, The ESOP must own at least 30% of the corporation’s stock immediately after the sale.

A

True.

59
Q

T/ F

To qualify for non-recognition of gain treatment, the corporation that establishes the ESOP must have no class of stock outstanding that is tradable on an established securities market.

A

True.

60
Q

Which of the following statements is (are) true?

I. Participants of a stock-bonus plan sponsored by a C corporation must be given pass-through voting rights for the employer stock held by the plan.
II. One disadvantage of an ESOP is that it may place an unnecessary cash-flow burden on the plan sponsor.

A

Both I and II

61
Q

In which of the following plans may an employer make tax-deductible contributions to a trust in the form of both principal and interest for a loan?

A) A stock bonus plan
B) An ESOP
C) A leveraged ESOP
D) An S corporation ESOP

A

C

Only a leveraged ESOP will have a loan and thus have principal and interest payments.

62
Q

T/ F

Some conflicts of interests among employees, the corporation, and management of the corporation do not require the ESOP trustee to resign.

A

True

63
Q

T/F

The trust of an ESOP is prohibited from borrowing money from a bank to purchase the employer stock.

A

False.

A key characteristic of the ESOP is that the trust may borrow money to purchase the employer stock.

64
Q

When calculating the plan contribution in a given year for a Defined Benefit Plan, an actuary might decrease the proposed funding amount based on which of the following factors:

A) Lower than expected plan forfeitures
B) Investment returns that are lower than anticipated
C) Lower employee turnover
D) A decrease in expected inflation

A

D

All the others would have the opposite effect on plan funding.

65
Q

What is the maximum loan you can take against your qualified plan?

A

The lesser of $50,000 or 50% of the vested account balance.

66
Q

Jim, aged 51, made a $100,000 direct rollover from the after-tax portion of his workplace profit-sharing account into a Roth IRA. He had an adjusted basis in the after-tax account of $150,000. The fair market value of the account at the time of the rollover was $500,000. Calculate the taxable amount of the Roth conversion and any applicable penalty.

A) $100,000 taxable, $10,000 tax penalty
B) $70,000 taxable, $0 tax penalty
C) $70,000 taxable, $7,000 tax penalty
D) $30,000 taxable, $0 tax penalty

A

The correct answer is (B).
Direct rollovers are exempt from the section 72(t) early withdrawal penalty tax, so the penalty must be $0. To calculate the amount of the Roth conversion contribution that is tax-free, the adjusted basis in the plan is divided by the fair market value of the plan as of the day of the distribution. This ratio is then multiplied by the gross rollover amount. Thus $30,000 of the $100,000 distribution is the return of the adjusted taxable basis: ($150,000 / $500,000) × $100,000. Accordingly, $70,000 ($100,000 − $30,000) will be subject to income tax.

67
Q

Perry operates In-N-Out Pharmacy, a sole proprietorship. In-N-Out sponsors a profit-sharing plan. Perry had net income of $205,000 and ½ of his self employment taxes was $15,000 for the year. If Perry makes a 25% of salary contribution on behalf of all of his employees to the profit-sharing plan, how much is the contribution to the profit-sharing plan on behalf of Perry?

A

$38,000

$205,000 - $15,000 (1/2 SE Tax)
= $190,000 Net SE Income
x.2 (.25/1.25)
= $38,000

68
Q

Allison Quinn, aged 43 earns an annual salary of $95,000. The company is implementing a group term-life-insurance plan and will pay the premium for an amount of coverage equal to her annual salary. What are the tax implications of this benefit for Allison, assuming 12 months of coverage are provided in the current year?

Assume it costs $0.10 / $1,000 of coverage.

A

$54

$95,000 - $50,000 (tax free benefit amount)
= $45,000 /$1,000
= $45 * 0.1
= $4.50 * 12
= $54

69
Q

Which of the following applies to key-person life insurance?

A) The premiums are tax-deductible.
B) The employee is the beneficiary.
C) The employer pays the cost.
D) The employee is the policy owner.

A

C

The premiums are not tax-deductible. The employer is the owner, premium payer and beneficiary.

70
Q

Wallace and Associates is considering implementing a buy/sell agreement in which each partner purchases a life insurance policy on each of the other partners. Which one of the following statements is correct, given this information?

A) The partners are entering into an entity redemption agreement.
B) Upon the death of an owner, the life insurance proceeds will be used to buy out the decedent’s share of the partnership. Those life insurance proceeds are taxable as ordinary income.
C) The amount of insurance per policy will equal the value of the partnership.
D) The partners are entering into a cross-purchase agreement.

A

D

Statement (A) is incorrect because the entity approach occurs when the business entity purchases life insurance policies on each owner. Statement (B) is incorrect because the life insurance proceeds are tax-exempt. Statement (C) is incorrect because each policy will be for the deceased partner’s share.

71
Q

Which of the following statements concerning HSAs is correct?

A) HSA distributions that are used to pay for non-medical expenses are subject to a 20% penalty.
B) Any HSA distributions not used for medical expenses are subject to long-term capital gains tax rates.
C) If HSA contributions are less than the prescribed limits, the account earnings are included in the account holder’s gross income for federal income tax purposes.
D) A married couple may contribute to up to one medical FSA and up to one HSA in any given year.

A

The correct answer is (A).
HSA distributions used to pay for non-medical expenses are subject to a 20% penalty as well as federal income tax (not, as statement B states, long-term capital gains taxes). Statement (C) is incorrect because, under the circumstances described, the account earnings would not be included in the account holder’s gross income for federal income tax purposes. Statement (D) is incorrect because if one spouse is covered by an FSA, this is treated as both spouses are being covered. If eligible, a married couple must select between an FSA and an HSA. An exception is if the FSA is “limited purpose” (covers dental and vision only)

72
Q

Mary is interested in installing a qualified plan for her employees and herself in her travel agency. She likes the idea of allowing herself and her employees maximum flexibility with regards to being able to access plan assets without incurring heavy taxes or penalties in case of need or hardship. Given her preferences, which qualified plan option is most appropriate?

A) Money Purchase Pension Plan
B) Profit-Sharing Plan/401(k) Plan
C) Cash Balance Plan
D) 403(b) Plan

A

The correct answer is (B).

Mary is interested in installing a qualified plan so that immediately erases answer choice (D) as an option, since 403(b)’s are not considered qualified plans. Choices (A) and (C) are pension type plans that limit withdrawals and do not normally provide loan options for participants.

73
Q

T/ F

All pension plans can skew benefits to longer serving employees.

A

False.

Only defined benefit pension plans can account for prior service at plan inception.

74
Q

T/F

Regarding life insurance in qualified plans, universal life insurance premiums cannot exceed 25% of the sponsor’s aggregate contribution to a participant’s defined contribution account.

A

True

75
Q

T/F

Regarding life insurance in qualified plans, there is no basis accoumulation for any Table 2001 amounts imputed to a plan participant for life insurance held by a qualified plan for their benefit.

A

False.

There is basis accumulation for any Table 2001 amounts paid for the cost of pure term insurance by a plan participant.

76
Q

Which of the following are characteristics of all profit-sharing plans?

A) They require annual plan contributions.
B) Compared to other qualified plans, they provide employee participants the greatest certainty when funding their own retirement.
C) The amount that an employer does or doesn’t contribute to a profit-sharing plan is based solely on the plan document filed with the IRS.
D) They can allow for distribution of stock or cash, depending on the complexion and requirements of the plan.

A

The correct answer is (D).

Statement (A) is incorrect because neither employees nor employers are required to make annual contributions. Statement (B) is incorrect because defined benefit plans provide far more certainty in regards to retirement funding. (C) is incorrect in that the amount an employer does or doesn’t contribute to a profit-sharing plan is a business decision and not based on the plan document. The formula delineating how much will be pares out to the participant’s account, will be included in the plan documents.

77
Q

John would like to offer his 35 employees some form of retirement plan. His concerns in offering a plan include cost, administrative complexity, and mandatory funding requirements. John would like to be able to have a vesting period as long as 6 years, the ability to have flexibility in funding, and have employee deferrals available in the plan. Which of the following plans should John consider to best address his concerns?

A) A Money Purchase plan
B) A Stock Bonus plan
C) A Cash Balance plan
D) A 401(k) plan

A

The correct answer is (B).

Answer choices (A), (C), and (D) have higher costs, greater administrative expenses and less funding flexibility than a Stock Bonus plan.

78
Q

Reggie has attained 2 years of service with his employer, Ink, Inc., (INK). INK sponsors a profit-sharing plan that provides for loans, and Reggie’s account balance within the plan is $150,000. He has not contributed any money to the profit-sharing plan. If the plan follows the least generous graded vesting schedule permitted under IRS guidelines, what is the maximum loan Reggie can take considering that he has not previously taken any plan loan?

A) $15,000
B) $30,000
C) $75,000
D) $150,000

A

The correct answer is (A).

The maximum loan permissible is the lesser of 50% of the participant’s vested account balance or $50,000, reduced by the highest outstanding loan balance within the 12 months prior to taking the new plan loan. In this case, Thomas is 20% vested in his profit-sharing plan account balance because he has only attained 2 years of service with the organization. (The least generous graded vesting schedule permitted would be a 2-to-6 year graded vesting schedule.) Since he is only vested in 20% of the account balance ($30,000), 50% of that vested account balance would be $15,000. Since Thomas has not taken any previous loans, his maximum loan would not be adjusted any further.

79
Q

Lynn, aged 42, has an IRA with an account balance of $200,000 and adjusted gross income (AGI) will be $80,000 this year. She has recently been declared disabled. According to this information, which of the following statements is (are) true?

I. She can immediately borrow up to $50,000 from her IRA account and repay the loan within 5 years.

II. If she takes a $50,000 distribution from her IRA, only a portion would be subject to the 10% early withdrawal penalty.

A

Neither.

Statement I is incorrect because loans are not permitted from IRAs. Statement II is incorrect because her declared disability permits her to avoid the penalty on the entire distribution.

80
Q

Jamie has a traditional IRA with $15,000 of nondeductible contributions and $5,000 of earnings. If she converts her traditional IRA to a Roth IRA, what amount of income will be included in her gross income?

A

$5,000

A conversion of a traditional IRA to a Roth IRA requires that the taxpayer include the deferred income in the IRA as taxable income in the year of the conversion.

81
Q

T/ F

To be eligible for a SIMPLE IRA you must be 21.

A

True

82
Q

When considering a Simple 401(k) plan, which of the following statements is true?

I. This type of plan can be better structured to maximize benefits for the owner.
II. This type of plan can be structured to maximize benefits for the owner’s spouse if employed by the sponsoring organization.

A

Both

Single participant plans, like Simple 401(k)s, can be structured to benefit both the owner and owner’s spouse.

83
Q

T/F

Assets in a rabbi trust are protected from sponsoring entities’ creditors.

A

False.

Rabbi trusts only protect against a change of heart of the sponsor not from insolvency. A secular trust would protect against insolvency claims, but could cause immediate taxation to the employee depending on the type of secular trust that was employed.

84
Q

Ron was awarded 5,000 shares of restricted stock at a time when the stock price was $10. Assume Ron properly makes an 83(b) election at the date of the award. The stock vests 5 years later at a price of $20, and he sells it then. What are the tax consequences in the year he makes the 83(b) election?

A) He has W-2 income of $50,000.
B) He has a long-term capital gain of $50,000.
C) He has W-2 income of $10,000.
D) He has a long-term capital gain of $10,000.

A

The correct answer is (A).

At the time Ron makes the 83(b) election, the value of the stock at that date will be included in his taxable income. Thus, Ron will have W-2 income of $50,000 (that is, $10 × 5,000).

85
Q

T/F

Regarding a non-qualified deferred compensation plan, there is an unfunded promise to pay.

A

True.

86
Q

Lauren received nonqualified stock options (NQSOs) with an exercise price equal to the fair market value (FMV) of $25 at the date of the grant. She exercised the options 3 years after the grant date, when the FMV of the stock was $30. Lauren then sells the stock for $35 3 years after exercising. Which of the following statements is true?

A) At the date of the grant, she will have ordinary income of $25.
B) At the date of exercise, she will have W-2 income of $30.
C) At the date of the sale, she will have W-2 income of $35.
D) At the date of sale, she will have long-term capital-gain of $5.

A

The correct answer is (D).

Lauren would not have any taxable income at the date of grant provided the exercise price was equal to the fair market value of the stock. Her long-term capital gain is $5, calculated as the sales price of $35 less the exercise price of $30.

87
Q

Barbie, aged 45, is a highly compensated employee of Tarjay, Inc., and she participates in their nonqualified deferred-compensation program. The year prior to this, she elects to have 10% of her $300,000 annual salary (or $30,000) deferred into the nonqualified plan, which is held in a secular trust with no vesting requirement. Her distribution election is to receive her deferred compensation when she turns 50 years old. Her contribution will be informally funded in a mutual fund account that is expected to increase to $50,000 at age 50. Based on this information, how much of Barbie’s compensation will be subject to ordinary income rates in this year?

A

$300,000

Because a secular trust with no vesting schedule is being used, there is no substantial risk of forfeiture. Therefore, none of the $30,000 contribution would be deferred for income tax purposes this year. Instead, the entire $300,000 of compensation would be subject to ordinary income tax rates this year.

88
Q

When considering benefit payouts to participants in a failed plan administered by the Pension Benefit Guaranty Corporation (PBGC), which of the following would be taken into consideration?

What are the other two things taken into account in addition to the correct answer?

A) whether the participant was a key employee.
B) the participant’s health status.
C) the participant’s ownership status.
D) the participant’s age.

A

The correct answer is (D).

The payout type, participant’s age, and degree to which the terminated plan was underfunded all factor into the benefit calculation.